Which proportion would you use to find what percent 6 is of 40?

Answers

Answer 1

Answer:

6/100 = x/40

Step-by-step explanation:

6% is represented by 6/100, x is the variable in the proportion, and 40 is the whole you are finding the percentage of


Related Questions

what is the sum of the first 5 terms of a geometric series with a sub 1 = 10 and r = 1/5?

Answers

Answer:

12.496

Step-by-step explanation:

a=10 (first term)

r= 1/5 (common ratio)

sum of first five terms

[tex] s_{5} = \frac{a( 1 - {r}^{n} ) }{1- r} [/tex]

=10{1-⅕⁵} ÷ 1-⅕

=10{1 - 1/3125} ÷ 4/5

=10{3124/3125} ÷ 4/5

=9.9968÷0.8

=12.496.

OR

a+ar+ar²+ar³+ar⁴

=10+(10x⅕)+(10x⅕²)+(10x⅕³)+(10x⅕⁴)

=10+2+0.4+0.08+0.016

=12+0.48+0.016

=12.496

HELP PLEASE HURRY


THANKS

Answers

Answer:

Step-by-step explanation:

What is the following product?

Answers

Answer:

D

Step-by-step explanation:

(5√2 - 4√3)^2 =

50 + 48 - 40√6 = 98 - 40√6

Given TVW,MZV = 90°, TV = 14, and VW= 9,
which of the following choices is used to find mZW?

Answers

here's the answer to your question

A civil servant had a salary of Rs 9600. Tax up to Rs 85000 per year would be exempted. If he paid Rs 4530 as a tax, how much percent of tax was imposed?

Answers

Answer:

4%

Step-by-step explanation:

Yearly salary = 9600 * 12 = Rs. 115200

Tax = Rs . 4530

[tex]Tax \ percent = \frac{4530}{115200}*100[/tex]

                    = 3.93 = 4%

Which angles are supplementary to each other?

• Angle 7 and Angle 3
• Angle 9 and Angle 12
• Angle 6 and Angle 12
• Angle 12 and Angle 10​

Answers

Answer:

∠9 and ∠12

Step-by-step explanation:

Supplementary angles are two angles that add up to 180. Looking at all the answer choices, A, C, and D have either Big-Big angle or Small-Small angle. What we need is Big-Small angle that is next to eachother, which is ∠9 and ∠12.

Hope this helped.

(Also sorry!)

Write 42 as a product of primes use index notation

Answers

The only way to write 42 as the product of primes (except to change the order of the factors) is 2 × 3 × 7. We call 2 × 3 × 7 the prime factorization of 42.

Calculate the axis of symmetry in the equation Y = (x+1) (x-4)

Come on anyone please helppppppp.

Answers

Answer:

Every parabola has a turning point called the axis of symmetry,so for this question you just have to find the turning points

x turning point= -b/2a,and the y turning point is gotten by replacing the value of x in the equation

y=(x+1)(x-4)

=x^2-3x-4

a is 1,b is -3 and c is -4

x= -(-3)/2(1)

=1.5

and y=x^2-3x-4

=1.5^2-3(1.5)-4

= -6.25

therefore the axis of symmetry is (1.5,-6.25)

I hope this helps

How much water should be added to 90ml of 45 percent acid solution to dilute it to 25 percent acid solution

Answers

Answer:

72 ml

Step-by-step explanation:

Let

x = amount of water that should be added

45% acid solution = 45/100 = 0.45

25% acid solution = 25/100 = 0.25

The equation is

90(0.45) = 0.25(x + 90)

40.5 = 0.25x + 22.5

40.5 - 22.5 = 0.25x

18 = 0.25x

Divide both sides by 0.25

x = 18/0.25

= 72

x = 72 ml

Which of the following are not Polynomials?

Answers

Answer:

E. is not a polynomial

Step-by-step explanation:

Because the degree of x is 2/5 which is not a polynomial.

Answer:

hmmm....

I don't think any of them are ????

[tex]\sqrt{2} = 2 ^{1/2}[/tex] that is a exponent that is a fraction ......

it appears that all the answers have fractional exponents, negative exponents, or divide by a variable??

Step-by-step explanation:

Polynomials cannot contain division by a variable.

For example, 2y2+7x/4 is a polynomial because 4 is not a variable. However, 2y2+7x/(1+x) is not a polynomial as it contains division by a variable.

Polynomials cannot contain negative exponents.

You cannot have 2y-2+7x-4. Negative exponents are a form of division by a variable (to make the negative exponent positive, you have to divide.) For example, x-3 is the same thing as 1/x3.

Polynomials cannot contain fractional exponents.

Terms containing fractional exponents (such as 3x+2y1/2-1) are not considered polynomials.

What is the rate of change of the amount earned with respect to hours worked for this function?
A) 1/13 hours per dollar
B) 2/5 hours per dollar
C) 3 dollars per hour
D) 13 dollars per hour

Answers

The rate of change of the amount earned with respect to hours worked for this function is D. 13 dollars per hour.

What is rate of change?

The rate of change for the point defined on the graph can be deduced by calculating the ratio of the difference between the two coordinate points given, similar to calculating the gradient or slope :

Point a = (5, 65) = (x₂, y₂)

Point b = (2, 26) = (x₁, y₁)

Calculating the gradient or slope :

Rate of change = (y₂ - y₁) / (x₂ - x₁)

Rate of change = (65 - 26) / (5 - 2)

Rate of change = 39 / 3

Rate= 13 dollars per hour

Learn more about rate of change on:

https://brainly.com/question/8728504

#SPJ1

Complete question

The graph shows the amount of money Miguel earns after working x hours. What is the rate of change of the amount earned with respect to hours worked for this function? hours per dollar hours per dollar 3 dollars per hour 13 dollars per hour

Complete the similarity for the two triangles shown.

Triangle ABC

Answers

Answer:

triangle EFD

Step-by-step explanation:

The difference in hour between full-time and part-time who work 5 hours a day is 2 working hours. How many hours per day do full-timers work?

Answers

Answer:

7

Step-by-step explanation:

The part timer works 5 hours a day

the difference between the part timer and fill time is 2

5+2=7

Please hurry I will mark you brainliest

The length of a rectangle is 5 less than 4 times the width. The perimeter is 136 m. Determine the dimensions of the rectangle. Write an equation and the “let statements”

Answers

Answer:

l = 14.6 m

b = 53.4 m

Step-by-step explanation:

Let breadth be x

Let lenght be 4x-5

so,

perimeter = 2(l+b)

136 = 2(4x-5+x)

or, 136 = 2(5x-5)

or, 136 = 10x - 10

or, 136 + 10 = 10x

or, 146 = 10x

or, x = 146/10

so, x = 14.6 m

so, breadth(x) = 14.6

so, lenght = 4x - 5

= (4×14.6)-5r

= 53.4m

HELPPP ME OUTTTT!!!!!

Answers

Answer:

Measure of the unknown angle is 21°.

Step-by-step explanation:

In the given question,

Measure of opposite side and the measure of hypotenuse have been given.

Therefore, we will use the trigonometric ratio for the unknown angle which has the ratio of opposite side and the hypotenuse.

[tex]\text{sin}\theta=\frac{\text{Opposite side}}{\text{Hypotenuse}}[/tex]

      [tex]=\frac{13}{36}[/tex]

      [tex]=0.36111[/tex]

[tex]\theta \approx \text{sin}^{-1}(0.36111)[/tex]

[tex]\theta \approx 21.17[/tex]

[tex]\theta \approx 21^{\circ}[/tex]

Therefore, measure of the unknown angle is 21°.

Write an equation of a line containing a point (5, 5) and is perpendicular to
the line passing through (5, 3) and (10, 6).
A) X/4 + 3y = 10
B) -5 X + 3 Y = 40
C) 5x + 3 Y= 40
D) X + y = 0

Answers

Answer is D

Explain 729:33(-2)

The length of a rectangle is three times the width. The area of the rectangle is 108 sq. inches what’s the triangles width?

Answers

Answer:ÑÑÑÑÑÑÑÑÑññÑÑÑÑññññññÑÑÑÑÑÑÑÑÑñÑÑÑÑññ

Step-by-step explanation:

What type of function is represented by the following graph:
(a) Linear
(b) Quadratic
(c) Absolute Value
(d) Exponential

Answers

Answer:

b) quadratic

Step-by-step explanation:

Answer:

b. quadratic

Step-by-step explanation:

it makes the shape of a parabola

hope this helps

what is 8/9 divided 2/3

Answers

Answer:

4/3

Step-by-step explanation:

8/9 ÷ 2/3

Copy dot flip

8/9 * 3/2

Rewriting

8/2 * 3/9

4 * 1/3

4/3


A tanker company has
transported 3.788 x 107 tons
of pineapples during its first
ten years of operation and
expects that total amount to
increase by 25% during the
next ten years.
Which measure
represents the total number of
tons of pineapples that the
company expects to transport
during the next ten years?
A 28.788 X 107 tons
B 9.47 x 106 tons
C 4.735 X 107 tons
D 4.038 X 107 tons

Answers

Answer:

c

Step-by-step explanation:

107 x 3.778 x 125/100 = 4.735 x 1-7

Prove that the circumcenter of a triangle is equidistant from its vertices.

Answers

Answer:

Step-by-step explanation:

The circumcenter is equidistant from the three vertices of the triangle. From the figure shown, we will prove DA = DB = DC. 2) DA = DB, DC = DB(If a point is on the perp. bisector of a segment, it is equidistant from each endpoint of the segment.)

Answer the question based on the data in the two-way table.

Gender Grades
Below
Average Above
Average Total
Boy 14 23 37
Girl 16 22 38
Total 30 45 75

Which statement is true?
A.
P(boy|above average grades) = P(boy)
B.
P(above average grades|boy) = P(above average grades)
C.
P(boy|above average grades) P(above average grades)
D.
P(above average grades|boy) = P(boy)

Answers

The answer is probably c

This is a graph of the function g(x) =-3x+2. Determine the domain value when th
range value is -4.

Answers

Range = -4= g(x)

Therefore, g(x) = -3x+2

or, -4=3x +2

or, 3x= -4-2

or, 3x= -6

or, x= -6/3 = -2

OPTION A is the correct answer.

When the range value is -4 for the function g(x) = -3x + 2, the corresponding domain value is x = 2.

To find the domain value when the range value is -4 for the function

g(x) = -3x + 2, we need to solve for x when g(x) = -4.

Given: g(x) = -3x + 2

When the range value is -4, we have:

-3x + 2 = -4

Now, isolate x:

-3x = -4 - 2

-3x = -6

Now, divide by -3 to solve for x:

x = -6 / -3

x = 2

So, when the range value is -4 for the function g(x) = -3x + 2, the corresponding domain value is x = 2.

To know more about range:

https://brainly.com/question/29204101

#SPJ2

Are shape I and shape II similar? If so, give the dilation that proves they are similar. If not, explain why the shapes are not similar.

Answers

Answer:

The answer is "They are similar".

Step-by-step explanation:

They were comparable in this respect because both aspect ratios of the top triangle are one square more. The top triangle is equal to the base triangles if you remove one square away from the height and width.

Otherwise, we can say that it forms all different. The dilation factor which translates that bottom left point of shape I to form II is 2. But this does not map the other shape I vertices onto form II. There's, therefore, no dilation in form I of maps on form II.

help please summer school sucks!!!

Answers

Answer:

X =30

Step-by-step explanation:

= 60+90

=150

angle sum property

x+150=180

x=180- 150

x= 30

Answer:

Step-by-step explanation:

90 + 60 = 150

(right angles = 90)

There is 180 degrees in a triangle, therefore,

180 - 150 = 30

Therefore, x = 30

p.s. Are you good at history?

p.p.s I dont like summer school either =)

Solve the equation 3(2x+9)=30

Answers

Answer:

x=1/2

Step-by-step explanation:

3(2x+9) = 30

Multiply out the 3

6x + 27 = 30

Subtract 27 on both sides

6x = 3

Divide by 6 on both sides

x = 1/2

Answer:

x=1/2

Step-by-step explanation

Please help ASAP!!!
What is m

Answers

Answer:

∡A  =115°

as for your question ... m is asking for the  "m" (measure) of angle A

Step-by-step explanation:

Answer:

∠ A = 118°

Step-by-step explanation:

The sum of the 3 angles in a triangle = 180°

Sum the 3 angles and equate to 180

3x + 13 + x - 8 + x = 180, that is

5x + 5 = 180 ( subtract 5 from both sides )

5x = 175 ( divide both sides by 5 )

x = 35

Then

∠ A = 3x + 13 = 3(35) + 13 = 105 + 13 = 118°

find the area of the shaded region,(π=3.14).

plx help me​

Answers

Answer:

115.395 cm2

Step-by-step explanation:

The radius of the whole figure: 14 : 2 = 7 (cm)

The area of the whole figure: 7 x 7 x 3.14 = 153.86 (cm2)

The area of the unshaded region:  3.5 x 3.5 x 3.14 = 38.465 (cm2)

The area of the shaded region: 153.86 - 38.465 =  115.395 (cm2)

Answer: 115.395 cm2.

Hope it helps!

nolan uses 7 inches of string to make each bracelet. if nolan makes 3 bracelets, how many inches of string will he use?

Answers

Answer:

21 inches

Step-by-step explanation:

We can write a ratio to solve

7 inches        x inches

-------------  = -------------------

1 bracelet      3 bracelets

Using cross products

7*3 = 1x

21 = x

21 inches

Answer:

21 inches

Step-by-step

This can be solved two ways: addition or multiplication.

Addition: Since there are three bracelets you could add 7 three times

                 7+7+7

             = 14+7

             = 21

Multiplication: Simply multiply 7 x 3= 21

What is the value of c in the interval (5,8) guaranteed by Rolle's Theorem for the function g(x)=−7x3+91x2−280x−9? Note that g(5)=g(8)=−9. (Do not include "c=" in your answer.)

Answers

Answer:

[tex]\displaystyle c = \frac{20}{3}[/tex]

Step-by-step explanation:

According to Rolle's Theorem, if f(a) = f(b) in an interval [a, b], then there must exist at least one c within (a, b) such that f'(c) = 0.

We are given that g(5) = g(8) = -9. Then according to Rolle's Theorem, there must be a c in (5, 8) such that g'(c) = 0.

So, differentiate the function. We can take the derivative of both sides with respect to x:

[tex]\displaystyle g'(x) = \frac{d}{dx}\left[ -7x^3 +91x^2 -280x - 9\right][/tex]

Differentiate:

[tex]g'(x) = -21x^2+182x-280[/tex]

Let g'(x) = 0:

[tex]0 = -21x^2+182x-280[/tex]

Solve for x. First, divide everything by negative seven:

[tex]0=3x^2-26x+40[/tex]

Factor:

[tex]0=(x-2)(3x-20)[/tex]

Zero Product Property:

[tex]x-2=0 \text{ or } 3x-20=0[/tex]

Solve for each case. Hence:

[tex]\displaystyle x=2 \text{ or } x = \frac{20}{3}[/tex]

Since the first solution is not within our interval, we can ignore it.

Therefore:

[tex]\displaystyle c = \frac{20}{3}[/tex]

Other Questions
3 log2 (x+1) - 2 = 13 The number of organic compounds that have been reported in 2019 is more than 10 million. This is due to carbon's ability to bond to other carbon molecules, calledelectron affinity.ionic bonding.catenation.electronegativity. 2. Determine the measure of the angles indicated by letters. Justify your answers with theproperties or theorems you used. ASAP What occurs after the homologous pairs are separated?A. The sister chromatids separate.B. Spindle fibers attach to the chromosomes.C. Haploid cells are formed.D. The nuclear envelope reforms. Use the point-slope formula to determine the equation of the line that has a slope of 12 and passes through point (0, 0). 3,125 subtracted by what can give me 514 According to a study published in Conservation Letters,a research team led by biologist Brian MacKenzie ran computer models of the population dynamics of the bluefin tuna that suggest that,even if fishing were banned immediately,the population of bluefin in the Atlantic and Mediterranean oceans will most likely collapse.This is a serious issue for sushi lovers everywhere,and some sociologists call it: A) technological diffusion. B) ecoterrorism. C) a tragedy of the commons. D) a public goods dilemma. Two sides of a triangle have lengths 13 m and 19 m. The angle between them is increasing at a rate of 2/min. How fast is the length of the third side increasing when the angle between the sides of fixed length is 60? (Round your answer to three decimal places.) The author mentions the fire train to In measuring GDP as the sum of all incomes earned in production, we should include: purchases of used goods. purchases of used goods. American purchases of foreign output. American purchases of foreign output. wages and salaries. wages and salaries. For the reaction 2HNO3 + Mg(OH)2 + Mg(NO3)2 + 2H20, how manygrams of magnesium nitrate are produced from 3.54 mol of nitricacid, HNO3?O a. 220O b. 263O c. 1050O d. 175 abolt from the blue, PLS HELP ME ON THIS QUESTION I WILL MRK YOU AS BRAINLIEST IF YOU KNOW THE ANSWER!!Which of the following measures is a measure of spread?A. medianB. rangeC. modeD. mean Hi, Which option is correct?? 3a + 2 = 20 5(b+1) = 103 (2y - 3) - 2y = y-32+ (2+4p) =6pPlease answer these questions with steps please! F is on the bisector of angle BCD. Find the length of FD (with lines over FD) List some rights of children and explain each in brief. For each kilogram of a persons weight 2.5 milligrams of a drug is to be given. what dosage should be given to a child who weighs 84 pounds? Use the fact that 1 lb = 0.45 kg how many inches is 775 centimeters write the first 10 multiples of 2 and 3 and find LCM.